Prove $x^3+3y^3+9z^3-9xyz=1$ has infinity integers solutions!












0












$begingroup$


A question from Alibaba Global Mathematics Competition (number theory)



Prove $displaystyle x^{3} +3y^{3} +9z^{3} -9xyz=1$ has infinitely many integer solutions.



The hint for the question is to transform the left side to a complex polynomial.



I found that:



Set $lambda =e^{j2pi /3}$ then the equation can be transformed to:



$$ left( x+3^{1/3} y+3^{2/3} zright)left( x+3^{1/3} lambda y+3^{2/3} lambda ^{2} zright)left( x+3^{1/3} lambda ^{2} y+3^{2/3} lambda zright)=1$$



but I don't know how to continue.










share|cite|improve this question











$endgroup$








  • 1




    $begingroup$
    Is this an on-going contest?
    $endgroup$
    – Lord Shark the Unknown
    Dec 23 '18 at 9:47






  • 1




    $begingroup$
    @LordSharktheUnknown It was held in Middle November.
    $endgroup$
    – Tianlalu
    Dec 23 '18 at 13:26
















0












$begingroup$


A question from Alibaba Global Mathematics Competition (number theory)



Prove $displaystyle x^{3} +3y^{3} +9z^{3} -9xyz=1$ has infinitely many integer solutions.



The hint for the question is to transform the left side to a complex polynomial.



I found that:



Set $lambda =e^{j2pi /3}$ then the equation can be transformed to:



$$ left( x+3^{1/3} y+3^{2/3} zright)left( x+3^{1/3} lambda y+3^{2/3} lambda ^{2} zright)left( x+3^{1/3} lambda ^{2} y+3^{2/3} lambda zright)=1$$



but I don't know how to continue.










share|cite|improve this question











$endgroup$








  • 1




    $begingroup$
    Is this an on-going contest?
    $endgroup$
    – Lord Shark the Unknown
    Dec 23 '18 at 9:47






  • 1




    $begingroup$
    @LordSharktheUnknown It was held in Middle November.
    $endgroup$
    – Tianlalu
    Dec 23 '18 at 13:26














0












0








0


3



$begingroup$


A question from Alibaba Global Mathematics Competition (number theory)



Prove $displaystyle x^{3} +3y^{3} +9z^{3} -9xyz=1$ has infinitely many integer solutions.



The hint for the question is to transform the left side to a complex polynomial.



I found that:



Set $lambda =e^{j2pi /3}$ then the equation can be transformed to:



$$ left( x+3^{1/3} y+3^{2/3} zright)left( x+3^{1/3} lambda y+3^{2/3} lambda ^{2} zright)left( x+3^{1/3} lambda ^{2} y+3^{2/3} lambda zright)=1$$



but I don't know how to continue.










share|cite|improve this question











$endgroup$




A question from Alibaba Global Mathematics Competition (number theory)



Prove $displaystyle x^{3} +3y^{3} +9z^{3} -9xyz=1$ has infinitely many integer solutions.



The hint for the question is to transform the left side to a complex polynomial.



I found that:



Set $lambda =e^{j2pi /3}$ then the equation can be transformed to:



$$ left( x+3^{1/3} y+3^{2/3} zright)left( x+3^{1/3} lambda y+3^{2/3} lambda ^{2} zright)left( x+3^{1/3} lambda ^{2} y+3^{2/3} lambda zright)=1$$



but I don't know how to continue.







number-theory contest-math






share|cite|improve this question















share|cite|improve this question













share|cite|improve this question




share|cite|improve this question








edited Dec 23 '18 at 9:39









Tianlalu

3,08421138




3,08421138










asked Dec 23 '18 at 9:07









tangyaotangyao

313




313








  • 1




    $begingroup$
    Is this an on-going contest?
    $endgroup$
    – Lord Shark the Unknown
    Dec 23 '18 at 9:47






  • 1




    $begingroup$
    @LordSharktheUnknown It was held in Middle November.
    $endgroup$
    – Tianlalu
    Dec 23 '18 at 13:26














  • 1




    $begingroup$
    Is this an on-going contest?
    $endgroup$
    – Lord Shark the Unknown
    Dec 23 '18 at 9:47






  • 1




    $begingroup$
    @LordSharktheUnknown It was held in Middle November.
    $endgroup$
    – Tianlalu
    Dec 23 '18 at 13:26








1




1




$begingroup$
Is this an on-going contest?
$endgroup$
– Lord Shark the Unknown
Dec 23 '18 at 9:47




$begingroup$
Is this an on-going contest?
$endgroup$
– Lord Shark the Unknown
Dec 23 '18 at 9:47




1




1




$begingroup$
@LordSharktheUnknown It was held in Middle November.
$endgroup$
– Tianlalu
Dec 23 '18 at 13:26




$begingroup$
@LordSharktheUnknown It was held in Middle November.
$endgroup$
– Tianlalu
Dec 23 '18 at 13:26










4 Answers
4






active

oldest

votes


















2












$begingroup$

this is my anser



first suppose we have an answer x,y,z , for simplicity,



set $displaystyle a=x,b=3^{1/3} y ,c=3^{2/3} z$



and take both sides pow3 ,we get



$displaystyle ( a+b+c)^{3}left( a+lambda b+lambda ^{2} cright)^{3}left( b+lambda ^{2} b+lambda cright)^{3}$=1





$displaystyle begin{array}{{>{displaystyle}l}}
left( a^{3} +b^{3} +c^{3} +3a^{2} b+3ab^{2} +3a^{2} c+3ac^{2} +3b^{2} c+3bc^{2} +6abcright) *\
left( a^{3} +b^{3} +c^{3} +3lambda a^{2} b+3lambda ^{2} ab^{2} +3lambda ^{2} a^{2} c+3lambda ac^{2} +3lambda b^{2} c+3lambda ^{2} bc^{2} +6abcright) *\
left( a^{3} +b^{3} +c^{3} +3lambda ^{2} a^{2} b+3lambda ab^{2} +3lambda a^{2} c+3lambda ^{2} ac^{2} +3lambda ^{2} b^{2} c+3lambda bc^{2} +6abcright) =1
end{array}$



$displaystyle begin{array}{{>{displaystyle}l}}
set A=a^{3} +b^{3} +c^{3} +6abc , B=3a^{2} b+3b^{2} c+3ac^{2} ,C=3ab^{2} +3a^{2} c+3bc^{2} , we get\
( A+B+C)left( A+lambda B+lambda ^{2} Cright)left( A+lambda ^{2} B+lambda Cright) =1\
set A'=A , B'=B/left( 3^{1/3}right) , C’=C/left( 3^{2/3}right)\
we get\
\
left( A'+3^{1/3} B'+3^{2/3} C'right)left( A'+3^{1/3} lambda B'+3^{2/3} lambda C'right)left( A'+3^{1/3} lambda ^{2} B'+3^{2/3} lambda C'right) =1\
\
then:\
A' =a^{3} +b^{3} +c^{3} +6abc is interger\
B'=B/left( 3^{1/3}right) =left( 3a^{2} b+3b^{2} c+3ac^{2}right) /3^{1/3} =3x^{2} y+9y^{2} z+9xz^{2} is interger\
C'=left( 3ab^{2} +3a^{2} c+3bc^{2}right) /3^{2/3} =3xy^{2} +3x^{2} z+9yz^{2} is interger\
\
so we get A^{prime 3} +3B^{prime 3} +9C^{prime 3} -9A'B'C'=1 such that A' >a,B' >b,C' >c\
we can repeat the process to get infinity solutions!
end{array}$






share|cite|improve this answer









$endgroup$





















    2












    $begingroup$

    First we find a non-trivial solution $(-2,0,1)$, ($(1,0,0)$ is trivial).




    Claim: Let $alpha=-2+0cdot 3^{frac13}+1cdot3^{frac23}$ and for any $ninBbb N$,
    $$alpha^n=x_n+y_n3^{frac13}+z_ncdot3^{frac23},$$
    then $(x_n,y_n,z_n)$ are solutions of the given equation.




    Proof: The idea is similar to finding general solutions for Pell's equation, but with a little Galois theory (Pell's equation corresponds to a field extension with degree $2$, which must be a Galois extension).



    The hint gives
    $$ left( x+ay+a^2 zright)left( x+a lambda y +a^2 lambda ^{2} zright)left( x+a lambda^2 y+a^2 lambda zright)=1$$
    with $a=3^{1/3}$ and $lambda =e^{j2pi /3}$. Take both sides to the power of $n$.



    Assume $(p,q,r)inBbb Z^3$ is a solution, then $( p+aq+a^2 r)$, $( p+a lambda q +a^2 lambda^2 r)$ and $(p+a lambda^2 q+a^2 lambda r)$ are conjugate elements in the splitting field. Since $Bbb Q(a,lambda)/Bbb Q$ is a Galois extension with degree $6$, any permutations between conjugate elements are isomorphic.



    So $(p+aq+a^2 r)^n$, $(p+a lambda q +a^2 lambda^2 r)^n$ and $(p+a lambda^2 q+a^2 lambda r)^n$ are also conjugate of each other.



    The first factor $(p+aq+a^2 r)^n$ is real so it corresponds to solutions.






    share|cite|improve this answer











    $endgroup$













    • $begingroup$
      I have find the anser 1 minutes ago!
      $endgroup$
      – tangyao
      Dec 23 '18 at 13:26










    • $begingroup$
      so many master here, i am lucky!
      $endgroup$
      – tangyao
      Dec 24 '18 at 15:26



















    2












    $begingroup$

    We get a fairly clean appearance by introducing the matrix
    $$
    M =
    left(
    begin{array}{ccc}
    0& beta &0 \
    0&0& beta \
    beta &0&0
    end{array}
    right)
    $$

    with $$ beta^3 = 3, $$
    so that
    $$ M^3 = 3 I $$
    Next we make a ring over integers with elements
    $$ xI + yM + z M^2 $$
    so that your polynomial is
    $$ det left( xI + yM + z M^2right) $$
    If two elements of the ring have determinant one, so does their product. If the coefficients of both elements are positive, so are the coefficients of the product, and larger than either. We get a multiplication with elemnts binary quadratic forms,
    $$ left( xI + yM + z M^2right) left( uI + vM + w M^2right)= left( (xu+3yw+3zv)I + (xv+yu+3zw)M + (xw+yv+zu) M^2right) $$
    So, if I have two triples $(x,y,z)$ and $(u,v,w)$ that evaluate to $1,$ so does this product triple,
    $$ color{blue}{ (xu+3yw+3zv, xv+yu+3zw,xw+yv+zu) } $$
    Beginning with the solution triple $(4,3,2)$ we may get a new larger positive solution from a given column vector $(x,y,z)^T$ by multiplying it by
    $$
    W =
    left(
    begin{array}{ccc}
    4&6 &9 \
    3&4& 6 \
    2 &3&4
    end{array}
    right)
    $$

    We get the sequence
    $$ (1,0,0), ; ; (4,3,2), ; ; (52,36,25), ; ;(649,450,312), ; ;(8104,5619,3896), ; ; ldots $$



    There are also solutions with negative elements, examples
    $$ (4,3,-4), ; (1,-18,12) $$
    Indeed, we can simply begin with $(1,0,0)$ and keep multiplying by



    $$
    W^{-1} =
    left(
    begin{array}{ccc}
    -2&3 &0 \
    0&-2& 3 \
    1 &0&-2
    end{array}
    right)
    $$



    $$ (1,0,0), ; ; (-2,0,1), ; ; (4,3,-4), ; ;(1,-18,12), ; ;(-56,72,-23), ; ; ldots $$



    So far, the only solutions I get are the left hand column of some $W^n$ where $n$ is a positive or negative integer, or $0.$






    share|cite|improve this answer











    $endgroup$













    • $begingroup$
      so many master here, i am lucky!
      $endgroup$
      – tangyao
      Dec 24 '18 at 15:26



















    1












    $begingroup$

    This is a particular case of the so-called Mathews diophantine equation, 1889 (or sometimes "cubic Pell") $(M_{m,n})∶ x^3+my^3+m^2 z^3-3mxyz=n$, where $m,n in mathbf Z$. It can be put under a more tractable form by factorizing a certain « normic » expression in the ring $mathbf Z[mu,omega]$, where $omega$ is a primitive cubic root of $1$ and $mu$ is the real cubic root of $m$. Precisely, the splitting field of $X^3-m$ is $mathbf Q (mu,omega)$ whose Galois group is $S_3$, generated by $tau: mu to mu,omega to omega^2$, and $sigma: omega to omega, mu to omegamu$. If 𝒩 is the norm (= product of conjugates) in $mathbf Q(μ,ω)/mathbf Q(mu)$, then 𝒩$(x+yμ+zμ^2)=(x+yμ+zμ^2)(x+yωμ+zω^2 μ^2)(x+yω^2 μ+zωμ^2)=$ $x^3+my^3+m^2 z^3-3mxyz$ (using $1+ω+ω^2=0$), hence eq. $(M_{m,n})$ is simply equivalent to 𝒩$(x+yμ+zμ^2)=n$. The function 𝒩 being multiplicative, we are reduced to solving the case $(M_{m,p})$, where $p$ is a prime number, and the case $(M_{m,pm 1})$.



    Your problem belongs to the case to $(M_{m,pm 1})$, with $m=3$. The equation 𝒩$(x+yμ+zμ^2)=pm 1$ means simply that $x+yμ+zμ^2 in mathbf Z[mu]$ is actually a unit. The ring of integers of $mathbf Q (mu)$ is classically known, see e.g. D. Marcus, "Number Fields", end of chap.2. In particular, for $m=3$, the ring of integers of $mathbf Q (mu)$ is $mathbf Z[mu]$ (which is a PID). We can apply Dirichlet's unit theorem, which states that the group of units has $mathbf Z$-rank $1$. The fundamental unit is determined in exercise 37 (c) of Marcus, chap.5. Note that the resolution of the eq. $(M_{m,pm 1})$ here is quite analogous to that of the quadratic Pell equation (cp. @Tianlalu).



    NB. To my knowledge, a complete solution of $(M_{m,p})$ is not available. A natural approach would be parallel to that exposed in the book of D. Cox, "Primes of the form $x^2+ny^2$", but this requires deep results from CFT.






    share|cite|improve this answer











    $endgroup$













    • $begingroup$
      so many master here, i am lucky!
      $endgroup$
      – tangyao
      Dec 24 '18 at 15:26











    Your Answer





    StackExchange.ifUsing("editor", function () {
    return StackExchange.using("mathjaxEditing", function () {
    StackExchange.MarkdownEditor.creationCallbacks.add(function (editor, postfix) {
    StackExchange.mathjaxEditing.prepareWmdForMathJax(editor, postfix, [["$", "$"], ["\\(","\\)"]]);
    });
    });
    }, "mathjax-editing");

    StackExchange.ready(function() {
    var channelOptions = {
    tags: "".split(" "),
    id: "69"
    };
    initTagRenderer("".split(" "), "".split(" "), channelOptions);

    StackExchange.using("externalEditor", function() {
    // Have to fire editor after snippets, if snippets enabled
    if (StackExchange.settings.snippets.snippetsEnabled) {
    StackExchange.using("snippets", function() {
    createEditor();
    });
    }
    else {
    createEditor();
    }
    });

    function createEditor() {
    StackExchange.prepareEditor({
    heartbeatType: 'answer',
    autoActivateHeartbeat: false,
    convertImagesToLinks: true,
    noModals: true,
    showLowRepImageUploadWarning: true,
    reputationToPostImages: 10,
    bindNavPrevention: true,
    postfix: "",
    imageUploader: {
    brandingHtml: "Powered by u003ca class="icon-imgur-white" href="https://imgur.com/"u003eu003c/au003e",
    contentPolicyHtml: "User contributions licensed under u003ca href="https://creativecommons.org/licenses/by-sa/3.0/"u003ecc by-sa 3.0 with attribution requiredu003c/au003e u003ca href="https://stackoverflow.com/legal/content-policy"u003e(content policy)u003c/au003e",
    allowUrls: true
    },
    noCode: true, onDemand: true,
    discardSelector: ".discard-answer"
    ,immediatelyShowMarkdownHelp:true
    });


    }
    });














    draft saved

    draft discarded


















    StackExchange.ready(
    function () {
    StackExchange.openid.initPostLogin('.new-post-login', 'https%3a%2f%2fmath.stackexchange.com%2fquestions%2f3050178%2fprove-x33y39z3-9xyz-1-has-infinity-integers-solutions%23new-answer', 'question_page');
    }
    );

    Post as a guest















    Required, but never shown

























    4 Answers
    4






    active

    oldest

    votes








    4 Answers
    4






    active

    oldest

    votes









    active

    oldest

    votes






    active

    oldest

    votes









    2












    $begingroup$

    this is my anser



    first suppose we have an answer x,y,z , for simplicity,



    set $displaystyle a=x,b=3^{1/3} y ,c=3^{2/3} z$



    and take both sides pow3 ,we get



    $displaystyle ( a+b+c)^{3}left( a+lambda b+lambda ^{2} cright)^{3}left( b+lambda ^{2} b+lambda cright)^{3}$=1





    $displaystyle begin{array}{{>{displaystyle}l}}
    left( a^{3} +b^{3} +c^{3} +3a^{2} b+3ab^{2} +3a^{2} c+3ac^{2} +3b^{2} c+3bc^{2} +6abcright) *\
    left( a^{3} +b^{3} +c^{3} +3lambda a^{2} b+3lambda ^{2} ab^{2} +3lambda ^{2} a^{2} c+3lambda ac^{2} +3lambda b^{2} c+3lambda ^{2} bc^{2} +6abcright) *\
    left( a^{3} +b^{3} +c^{3} +3lambda ^{2} a^{2} b+3lambda ab^{2} +3lambda a^{2} c+3lambda ^{2} ac^{2} +3lambda ^{2} b^{2} c+3lambda bc^{2} +6abcright) =1
    end{array}$



    $displaystyle begin{array}{{>{displaystyle}l}}
    set A=a^{3} +b^{3} +c^{3} +6abc , B=3a^{2} b+3b^{2} c+3ac^{2} ,C=3ab^{2} +3a^{2} c+3bc^{2} , we get\
    ( A+B+C)left( A+lambda B+lambda ^{2} Cright)left( A+lambda ^{2} B+lambda Cright) =1\
    set A'=A , B'=B/left( 3^{1/3}right) , C’=C/left( 3^{2/3}right)\
    we get\
    \
    left( A'+3^{1/3} B'+3^{2/3} C'right)left( A'+3^{1/3} lambda B'+3^{2/3} lambda C'right)left( A'+3^{1/3} lambda ^{2} B'+3^{2/3} lambda C'right) =1\
    \
    then:\
    A' =a^{3} +b^{3} +c^{3} +6abc is interger\
    B'=B/left( 3^{1/3}right) =left( 3a^{2} b+3b^{2} c+3ac^{2}right) /3^{1/3} =3x^{2} y+9y^{2} z+9xz^{2} is interger\
    C'=left( 3ab^{2} +3a^{2} c+3bc^{2}right) /3^{2/3} =3xy^{2} +3x^{2} z+9yz^{2} is interger\
    \
    so we get A^{prime 3} +3B^{prime 3} +9C^{prime 3} -9A'B'C'=1 such that A' >a,B' >b,C' >c\
    we can repeat the process to get infinity solutions!
    end{array}$






    share|cite|improve this answer









    $endgroup$


















      2












      $begingroup$

      this is my anser



      first suppose we have an answer x,y,z , for simplicity,



      set $displaystyle a=x,b=3^{1/3} y ,c=3^{2/3} z$



      and take both sides pow3 ,we get



      $displaystyle ( a+b+c)^{3}left( a+lambda b+lambda ^{2} cright)^{3}left( b+lambda ^{2} b+lambda cright)^{3}$=1





      $displaystyle begin{array}{{>{displaystyle}l}}
      left( a^{3} +b^{3} +c^{3} +3a^{2} b+3ab^{2} +3a^{2} c+3ac^{2} +3b^{2} c+3bc^{2} +6abcright) *\
      left( a^{3} +b^{3} +c^{3} +3lambda a^{2} b+3lambda ^{2} ab^{2} +3lambda ^{2} a^{2} c+3lambda ac^{2} +3lambda b^{2} c+3lambda ^{2} bc^{2} +6abcright) *\
      left( a^{3} +b^{3} +c^{3} +3lambda ^{2} a^{2} b+3lambda ab^{2} +3lambda a^{2} c+3lambda ^{2} ac^{2} +3lambda ^{2} b^{2} c+3lambda bc^{2} +6abcright) =1
      end{array}$



      $displaystyle begin{array}{{>{displaystyle}l}}
      set A=a^{3} +b^{3} +c^{3} +6abc , B=3a^{2} b+3b^{2} c+3ac^{2} ,C=3ab^{2} +3a^{2} c+3bc^{2} , we get\
      ( A+B+C)left( A+lambda B+lambda ^{2} Cright)left( A+lambda ^{2} B+lambda Cright) =1\
      set A'=A , B'=B/left( 3^{1/3}right) , C’=C/left( 3^{2/3}right)\
      we get\
      \
      left( A'+3^{1/3} B'+3^{2/3} C'right)left( A'+3^{1/3} lambda B'+3^{2/3} lambda C'right)left( A'+3^{1/3} lambda ^{2} B'+3^{2/3} lambda C'right) =1\
      \
      then:\
      A' =a^{3} +b^{3} +c^{3} +6abc is interger\
      B'=B/left( 3^{1/3}right) =left( 3a^{2} b+3b^{2} c+3ac^{2}right) /3^{1/3} =3x^{2} y+9y^{2} z+9xz^{2} is interger\
      C'=left( 3ab^{2} +3a^{2} c+3bc^{2}right) /3^{2/3} =3xy^{2} +3x^{2} z+9yz^{2} is interger\
      \
      so we get A^{prime 3} +3B^{prime 3} +9C^{prime 3} -9A'B'C'=1 such that A' >a,B' >b,C' >c\
      we can repeat the process to get infinity solutions!
      end{array}$






      share|cite|improve this answer









      $endgroup$
















        2












        2








        2





        $begingroup$

        this is my anser



        first suppose we have an answer x,y,z , for simplicity,



        set $displaystyle a=x,b=3^{1/3} y ,c=3^{2/3} z$



        and take both sides pow3 ,we get



        $displaystyle ( a+b+c)^{3}left( a+lambda b+lambda ^{2} cright)^{3}left( b+lambda ^{2} b+lambda cright)^{3}$=1





        $displaystyle begin{array}{{>{displaystyle}l}}
        left( a^{3} +b^{3} +c^{3} +3a^{2} b+3ab^{2} +3a^{2} c+3ac^{2} +3b^{2} c+3bc^{2} +6abcright) *\
        left( a^{3} +b^{3} +c^{3} +3lambda a^{2} b+3lambda ^{2} ab^{2} +3lambda ^{2} a^{2} c+3lambda ac^{2} +3lambda b^{2} c+3lambda ^{2} bc^{2} +6abcright) *\
        left( a^{3} +b^{3} +c^{3} +3lambda ^{2} a^{2} b+3lambda ab^{2} +3lambda a^{2} c+3lambda ^{2} ac^{2} +3lambda ^{2} b^{2} c+3lambda bc^{2} +6abcright) =1
        end{array}$



        $displaystyle begin{array}{{>{displaystyle}l}}
        set A=a^{3} +b^{3} +c^{3} +6abc , B=3a^{2} b+3b^{2} c+3ac^{2} ,C=3ab^{2} +3a^{2} c+3bc^{2} , we get\
        ( A+B+C)left( A+lambda B+lambda ^{2} Cright)left( A+lambda ^{2} B+lambda Cright) =1\
        set A'=A , B'=B/left( 3^{1/3}right) , C’=C/left( 3^{2/3}right)\
        we get\
        \
        left( A'+3^{1/3} B'+3^{2/3} C'right)left( A'+3^{1/3} lambda B'+3^{2/3} lambda C'right)left( A'+3^{1/3} lambda ^{2} B'+3^{2/3} lambda C'right) =1\
        \
        then:\
        A' =a^{3} +b^{3} +c^{3} +6abc is interger\
        B'=B/left( 3^{1/3}right) =left( 3a^{2} b+3b^{2} c+3ac^{2}right) /3^{1/3} =3x^{2} y+9y^{2} z+9xz^{2} is interger\
        C'=left( 3ab^{2} +3a^{2} c+3bc^{2}right) /3^{2/3} =3xy^{2} +3x^{2} z+9yz^{2} is interger\
        \
        so we get A^{prime 3} +3B^{prime 3} +9C^{prime 3} -9A'B'C'=1 such that A' >a,B' >b,C' >c\
        we can repeat the process to get infinity solutions!
        end{array}$






        share|cite|improve this answer









        $endgroup$



        this is my anser



        first suppose we have an answer x,y,z , for simplicity,



        set $displaystyle a=x,b=3^{1/3} y ,c=3^{2/3} z$



        and take both sides pow3 ,we get



        $displaystyle ( a+b+c)^{3}left( a+lambda b+lambda ^{2} cright)^{3}left( b+lambda ^{2} b+lambda cright)^{3}$=1





        $displaystyle begin{array}{{>{displaystyle}l}}
        left( a^{3} +b^{3} +c^{3} +3a^{2} b+3ab^{2} +3a^{2} c+3ac^{2} +3b^{2} c+3bc^{2} +6abcright) *\
        left( a^{3} +b^{3} +c^{3} +3lambda a^{2} b+3lambda ^{2} ab^{2} +3lambda ^{2} a^{2} c+3lambda ac^{2} +3lambda b^{2} c+3lambda ^{2} bc^{2} +6abcright) *\
        left( a^{3} +b^{3} +c^{3} +3lambda ^{2} a^{2} b+3lambda ab^{2} +3lambda a^{2} c+3lambda ^{2} ac^{2} +3lambda ^{2} b^{2} c+3lambda bc^{2} +6abcright) =1
        end{array}$



        $displaystyle begin{array}{{>{displaystyle}l}}
        set A=a^{3} +b^{3} +c^{3} +6abc , B=3a^{2} b+3b^{2} c+3ac^{2} ,C=3ab^{2} +3a^{2} c+3bc^{2} , we get\
        ( A+B+C)left( A+lambda B+lambda ^{2} Cright)left( A+lambda ^{2} B+lambda Cright) =1\
        set A'=A , B'=B/left( 3^{1/3}right) , C’=C/left( 3^{2/3}right)\
        we get\
        \
        left( A'+3^{1/3} B'+3^{2/3} C'right)left( A'+3^{1/3} lambda B'+3^{2/3} lambda C'right)left( A'+3^{1/3} lambda ^{2} B'+3^{2/3} lambda C'right) =1\
        \
        then:\
        A' =a^{3} +b^{3} +c^{3} +6abc is interger\
        B'=B/left( 3^{1/3}right) =left( 3a^{2} b+3b^{2} c+3ac^{2}right) /3^{1/3} =3x^{2} y+9y^{2} z+9xz^{2} is interger\
        C'=left( 3ab^{2} +3a^{2} c+3bc^{2}right) /3^{2/3} =3xy^{2} +3x^{2} z+9yz^{2} is interger\
        \
        so we get A^{prime 3} +3B^{prime 3} +9C^{prime 3} -9A'B'C'=1 such that A' >a,B' >b,C' >c\
        we can repeat the process to get infinity solutions!
        end{array}$







        share|cite|improve this answer












        share|cite|improve this answer



        share|cite|improve this answer










        answered Dec 23 '18 at 13:51









        tangyaotangyao

        313




        313























            2












            $begingroup$

            First we find a non-trivial solution $(-2,0,1)$, ($(1,0,0)$ is trivial).




            Claim: Let $alpha=-2+0cdot 3^{frac13}+1cdot3^{frac23}$ and for any $ninBbb N$,
            $$alpha^n=x_n+y_n3^{frac13}+z_ncdot3^{frac23},$$
            then $(x_n,y_n,z_n)$ are solutions of the given equation.




            Proof: The idea is similar to finding general solutions for Pell's equation, but with a little Galois theory (Pell's equation corresponds to a field extension with degree $2$, which must be a Galois extension).



            The hint gives
            $$ left( x+ay+a^2 zright)left( x+a lambda y +a^2 lambda ^{2} zright)left( x+a lambda^2 y+a^2 lambda zright)=1$$
            with $a=3^{1/3}$ and $lambda =e^{j2pi /3}$. Take both sides to the power of $n$.



            Assume $(p,q,r)inBbb Z^3$ is a solution, then $( p+aq+a^2 r)$, $( p+a lambda q +a^2 lambda^2 r)$ and $(p+a lambda^2 q+a^2 lambda r)$ are conjugate elements in the splitting field. Since $Bbb Q(a,lambda)/Bbb Q$ is a Galois extension with degree $6$, any permutations between conjugate elements are isomorphic.



            So $(p+aq+a^2 r)^n$, $(p+a lambda q +a^2 lambda^2 r)^n$ and $(p+a lambda^2 q+a^2 lambda r)^n$ are also conjugate of each other.



            The first factor $(p+aq+a^2 r)^n$ is real so it corresponds to solutions.






            share|cite|improve this answer











            $endgroup$













            • $begingroup$
              I have find the anser 1 minutes ago!
              $endgroup$
              – tangyao
              Dec 23 '18 at 13:26










            • $begingroup$
              so many master here, i am lucky!
              $endgroup$
              – tangyao
              Dec 24 '18 at 15:26
















            2












            $begingroup$

            First we find a non-trivial solution $(-2,0,1)$, ($(1,0,0)$ is trivial).




            Claim: Let $alpha=-2+0cdot 3^{frac13}+1cdot3^{frac23}$ and for any $ninBbb N$,
            $$alpha^n=x_n+y_n3^{frac13}+z_ncdot3^{frac23},$$
            then $(x_n,y_n,z_n)$ are solutions of the given equation.




            Proof: The idea is similar to finding general solutions for Pell's equation, but with a little Galois theory (Pell's equation corresponds to a field extension with degree $2$, which must be a Galois extension).



            The hint gives
            $$ left( x+ay+a^2 zright)left( x+a lambda y +a^2 lambda ^{2} zright)left( x+a lambda^2 y+a^2 lambda zright)=1$$
            with $a=3^{1/3}$ and $lambda =e^{j2pi /3}$. Take both sides to the power of $n$.



            Assume $(p,q,r)inBbb Z^3$ is a solution, then $( p+aq+a^2 r)$, $( p+a lambda q +a^2 lambda^2 r)$ and $(p+a lambda^2 q+a^2 lambda r)$ are conjugate elements in the splitting field. Since $Bbb Q(a,lambda)/Bbb Q$ is a Galois extension with degree $6$, any permutations between conjugate elements are isomorphic.



            So $(p+aq+a^2 r)^n$, $(p+a lambda q +a^2 lambda^2 r)^n$ and $(p+a lambda^2 q+a^2 lambda r)^n$ are also conjugate of each other.



            The first factor $(p+aq+a^2 r)^n$ is real so it corresponds to solutions.






            share|cite|improve this answer











            $endgroup$













            • $begingroup$
              I have find the anser 1 minutes ago!
              $endgroup$
              – tangyao
              Dec 23 '18 at 13:26










            • $begingroup$
              so many master here, i am lucky!
              $endgroup$
              – tangyao
              Dec 24 '18 at 15:26














            2












            2








            2





            $begingroup$

            First we find a non-trivial solution $(-2,0,1)$, ($(1,0,0)$ is trivial).




            Claim: Let $alpha=-2+0cdot 3^{frac13}+1cdot3^{frac23}$ and for any $ninBbb N$,
            $$alpha^n=x_n+y_n3^{frac13}+z_ncdot3^{frac23},$$
            then $(x_n,y_n,z_n)$ are solutions of the given equation.




            Proof: The idea is similar to finding general solutions for Pell's equation, but with a little Galois theory (Pell's equation corresponds to a field extension with degree $2$, which must be a Galois extension).



            The hint gives
            $$ left( x+ay+a^2 zright)left( x+a lambda y +a^2 lambda ^{2} zright)left( x+a lambda^2 y+a^2 lambda zright)=1$$
            with $a=3^{1/3}$ and $lambda =e^{j2pi /3}$. Take both sides to the power of $n$.



            Assume $(p,q,r)inBbb Z^3$ is a solution, then $( p+aq+a^2 r)$, $( p+a lambda q +a^2 lambda^2 r)$ and $(p+a lambda^2 q+a^2 lambda r)$ are conjugate elements in the splitting field. Since $Bbb Q(a,lambda)/Bbb Q$ is a Galois extension with degree $6$, any permutations between conjugate elements are isomorphic.



            So $(p+aq+a^2 r)^n$, $(p+a lambda q +a^2 lambda^2 r)^n$ and $(p+a lambda^2 q+a^2 lambda r)^n$ are also conjugate of each other.



            The first factor $(p+aq+a^2 r)^n$ is real so it corresponds to solutions.






            share|cite|improve this answer











            $endgroup$



            First we find a non-trivial solution $(-2,0,1)$, ($(1,0,0)$ is trivial).




            Claim: Let $alpha=-2+0cdot 3^{frac13}+1cdot3^{frac23}$ and for any $ninBbb N$,
            $$alpha^n=x_n+y_n3^{frac13}+z_ncdot3^{frac23},$$
            then $(x_n,y_n,z_n)$ are solutions of the given equation.




            Proof: The idea is similar to finding general solutions for Pell's equation, but with a little Galois theory (Pell's equation corresponds to a field extension with degree $2$, which must be a Galois extension).



            The hint gives
            $$ left( x+ay+a^2 zright)left( x+a lambda y +a^2 lambda ^{2} zright)left( x+a lambda^2 y+a^2 lambda zright)=1$$
            with $a=3^{1/3}$ and $lambda =e^{j2pi /3}$. Take both sides to the power of $n$.



            Assume $(p,q,r)inBbb Z^3$ is a solution, then $( p+aq+a^2 r)$, $( p+a lambda q +a^2 lambda^2 r)$ and $(p+a lambda^2 q+a^2 lambda r)$ are conjugate elements in the splitting field. Since $Bbb Q(a,lambda)/Bbb Q$ is a Galois extension with degree $6$, any permutations between conjugate elements are isomorphic.



            So $(p+aq+a^2 r)^n$, $(p+a lambda q +a^2 lambda^2 r)^n$ and $(p+a lambda^2 q+a^2 lambda r)^n$ are also conjugate of each other.



            The first factor $(p+aq+a^2 r)^n$ is real so it corresponds to solutions.







            share|cite|improve this answer














            share|cite|improve this answer



            share|cite|improve this answer








            edited Dec 23 '18 at 14:01

























            answered Dec 23 '18 at 13:15









            TianlaluTianlalu

            3,08421138




            3,08421138












            • $begingroup$
              I have find the anser 1 minutes ago!
              $endgroup$
              – tangyao
              Dec 23 '18 at 13:26










            • $begingroup$
              so many master here, i am lucky!
              $endgroup$
              – tangyao
              Dec 24 '18 at 15:26


















            • $begingroup$
              I have find the anser 1 minutes ago!
              $endgroup$
              – tangyao
              Dec 23 '18 at 13:26










            • $begingroup$
              so many master here, i am lucky!
              $endgroup$
              – tangyao
              Dec 24 '18 at 15:26
















            $begingroup$
            I have find the anser 1 minutes ago!
            $endgroup$
            – tangyao
            Dec 23 '18 at 13:26




            $begingroup$
            I have find the anser 1 minutes ago!
            $endgroup$
            – tangyao
            Dec 23 '18 at 13:26












            $begingroup$
            so many master here, i am lucky!
            $endgroup$
            – tangyao
            Dec 24 '18 at 15:26




            $begingroup$
            so many master here, i am lucky!
            $endgroup$
            – tangyao
            Dec 24 '18 at 15:26











            2












            $begingroup$

            We get a fairly clean appearance by introducing the matrix
            $$
            M =
            left(
            begin{array}{ccc}
            0& beta &0 \
            0&0& beta \
            beta &0&0
            end{array}
            right)
            $$

            with $$ beta^3 = 3, $$
            so that
            $$ M^3 = 3 I $$
            Next we make a ring over integers with elements
            $$ xI + yM + z M^2 $$
            so that your polynomial is
            $$ det left( xI + yM + z M^2right) $$
            If two elements of the ring have determinant one, so does their product. If the coefficients of both elements are positive, so are the coefficients of the product, and larger than either. We get a multiplication with elemnts binary quadratic forms,
            $$ left( xI + yM + z M^2right) left( uI + vM + w M^2right)= left( (xu+3yw+3zv)I + (xv+yu+3zw)M + (xw+yv+zu) M^2right) $$
            So, if I have two triples $(x,y,z)$ and $(u,v,w)$ that evaluate to $1,$ so does this product triple,
            $$ color{blue}{ (xu+3yw+3zv, xv+yu+3zw,xw+yv+zu) } $$
            Beginning with the solution triple $(4,3,2)$ we may get a new larger positive solution from a given column vector $(x,y,z)^T$ by multiplying it by
            $$
            W =
            left(
            begin{array}{ccc}
            4&6 &9 \
            3&4& 6 \
            2 &3&4
            end{array}
            right)
            $$

            We get the sequence
            $$ (1,0,0), ; ; (4,3,2), ; ; (52,36,25), ; ;(649,450,312), ; ;(8104,5619,3896), ; ; ldots $$



            There are also solutions with negative elements, examples
            $$ (4,3,-4), ; (1,-18,12) $$
            Indeed, we can simply begin with $(1,0,0)$ and keep multiplying by



            $$
            W^{-1} =
            left(
            begin{array}{ccc}
            -2&3 &0 \
            0&-2& 3 \
            1 &0&-2
            end{array}
            right)
            $$



            $$ (1,0,0), ; ; (-2,0,1), ; ; (4,3,-4), ; ;(1,-18,12), ; ;(-56,72,-23), ; ; ldots $$



            So far, the only solutions I get are the left hand column of some $W^n$ where $n$ is a positive or negative integer, or $0.$






            share|cite|improve this answer











            $endgroup$













            • $begingroup$
              so many master here, i am lucky!
              $endgroup$
              – tangyao
              Dec 24 '18 at 15:26
















            2












            $begingroup$

            We get a fairly clean appearance by introducing the matrix
            $$
            M =
            left(
            begin{array}{ccc}
            0& beta &0 \
            0&0& beta \
            beta &0&0
            end{array}
            right)
            $$

            with $$ beta^3 = 3, $$
            so that
            $$ M^3 = 3 I $$
            Next we make a ring over integers with elements
            $$ xI + yM + z M^2 $$
            so that your polynomial is
            $$ det left( xI + yM + z M^2right) $$
            If two elements of the ring have determinant one, so does their product. If the coefficients of both elements are positive, so are the coefficients of the product, and larger than either. We get a multiplication with elemnts binary quadratic forms,
            $$ left( xI + yM + z M^2right) left( uI + vM + w M^2right)= left( (xu+3yw+3zv)I + (xv+yu+3zw)M + (xw+yv+zu) M^2right) $$
            So, if I have two triples $(x,y,z)$ and $(u,v,w)$ that evaluate to $1,$ so does this product triple,
            $$ color{blue}{ (xu+3yw+3zv, xv+yu+3zw,xw+yv+zu) } $$
            Beginning with the solution triple $(4,3,2)$ we may get a new larger positive solution from a given column vector $(x,y,z)^T$ by multiplying it by
            $$
            W =
            left(
            begin{array}{ccc}
            4&6 &9 \
            3&4& 6 \
            2 &3&4
            end{array}
            right)
            $$

            We get the sequence
            $$ (1,0,0), ; ; (4,3,2), ; ; (52,36,25), ; ;(649,450,312), ; ;(8104,5619,3896), ; ; ldots $$



            There are also solutions with negative elements, examples
            $$ (4,3,-4), ; (1,-18,12) $$
            Indeed, we can simply begin with $(1,0,0)$ and keep multiplying by



            $$
            W^{-1} =
            left(
            begin{array}{ccc}
            -2&3 &0 \
            0&-2& 3 \
            1 &0&-2
            end{array}
            right)
            $$



            $$ (1,0,0), ; ; (-2,0,1), ; ; (4,3,-4), ; ;(1,-18,12), ; ;(-56,72,-23), ; ; ldots $$



            So far, the only solutions I get are the left hand column of some $W^n$ where $n$ is a positive or negative integer, or $0.$






            share|cite|improve this answer











            $endgroup$













            • $begingroup$
              so many master here, i am lucky!
              $endgroup$
              – tangyao
              Dec 24 '18 at 15:26














            2












            2








            2





            $begingroup$

            We get a fairly clean appearance by introducing the matrix
            $$
            M =
            left(
            begin{array}{ccc}
            0& beta &0 \
            0&0& beta \
            beta &0&0
            end{array}
            right)
            $$

            with $$ beta^3 = 3, $$
            so that
            $$ M^3 = 3 I $$
            Next we make a ring over integers with elements
            $$ xI + yM + z M^2 $$
            so that your polynomial is
            $$ det left( xI + yM + z M^2right) $$
            If two elements of the ring have determinant one, so does their product. If the coefficients of both elements are positive, so are the coefficients of the product, and larger than either. We get a multiplication with elemnts binary quadratic forms,
            $$ left( xI + yM + z M^2right) left( uI + vM + w M^2right)= left( (xu+3yw+3zv)I + (xv+yu+3zw)M + (xw+yv+zu) M^2right) $$
            So, if I have two triples $(x,y,z)$ and $(u,v,w)$ that evaluate to $1,$ so does this product triple,
            $$ color{blue}{ (xu+3yw+3zv, xv+yu+3zw,xw+yv+zu) } $$
            Beginning with the solution triple $(4,3,2)$ we may get a new larger positive solution from a given column vector $(x,y,z)^T$ by multiplying it by
            $$
            W =
            left(
            begin{array}{ccc}
            4&6 &9 \
            3&4& 6 \
            2 &3&4
            end{array}
            right)
            $$

            We get the sequence
            $$ (1,0,0), ; ; (4,3,2), ; ; (52,36,25), ; ;(649,450,312), ; ;(8104,5619,3896), ; ; ldots $$



            There are also solutions with negative elements, examples
            $$ (4,3,-4), ; (1,-18,12) $$
            Indeed, we can simply begin with $(1,0,0)$ and keep multiplying by



            $$
            W^{-1} =
            left(
            begin{array}{ccc}
            -2&3 &0 \
            0&-2& 3 \
            1 &0&-2
            end{array}
            right)
            $$



            $$ (1,0,0), ; ; (-2,0,1), ; ; (4,3,-4), ; ;(1,-18,12), ; ;(-56,72,-23), ; ; ldots $$



            So far, the only solutions I get are the left hand column of some $W^n$ where $n$ is a positive or negative integer, or $0.$






            share|cite|improve this answer











            $endgroup$



            We get a fairly clean appearance by introducing the matrix
            $$
            M =
            left(
            begin{array}{ccc}
            0& beta &0 \
            0&0& beta \
            beta &0&0
            end{array}
            right)
            $$

            with $$ beta^3 = 3, $$
            so that
            $$ M^3 = 3 I $$
            Next we make a ring over integers with elements
            $$ xI + yM + z M^2 $$
            so that your polynomial is
            $$ det left( xI + yM + z M^2right) $$
            If two elements of the ring have determinant one, so does their product. If the coefficients of both elements are positive, so are the coefficients of the product, and larger than either. We get a multiplication with elemnts binary quadratic forms,
            $$ left( xI + yM + z M^2right) left( uI + vM + w M^2right)= left( (xu+3yw+3zv)I + (xv+yu+3zw)M + (xw+yv+zu) M^2right) $$
            So, if I have two triples $(x,y,z)$ and $(u,v,w)$ that evaluate to $1,$ so does this product triple,
            $$ color{blue}{ (xu+3yw+3zv, xv+yu+3zw,xw+yv+zu) } $$
            Beginning with the solution triple $(4,3,2)$ we may get a new larger positive solution from a given column vector $(x,y,z)^T$ by multiplying it by
            $$
            W =
            left(
            begin{array}{ccc}
            4&6 &9 \
            3&4& 6 \
            2 &3&4
            end{array}
            right)
            $$

            We get the sequence
            $$ (1,0,0), ; ; (4,3,2), ; ; (52,36,25), ; ;(649,450,312), ; ;(8104,5619,3896), ; ; ldots $$



            There are also solutions with negative elements, examples
            $$ (4,3,-4), ; (1,-18,12) $$
            Indeed, we can simply begin with $(1,0,0)$ and keep multiplying by



            $$
            W^{-1} =
            left(
            begin{array}{ccc}
            -2&3 &0 \
            0&-2& 3 \
            1 &0&-2
            end{array}
            right)
            $$



            $$ (1,0,0), ; ; (-2,0,1), ; ; (4,3,-4), ; ;(1,-18,12), ; ;(-56,72,-23), ; ; ldots $$



            So far, the only solutions I get are the left hand column of some $W^n$ where $n$ is a positive or negative integer, or $0.$







            share|cite|improve this answer














            share|cite|improve this answer



            share|cite|improve this answer








            edited Dec 23 '18 at 16:43

























            answered Dec 23 '18 at 16:08









            Will JagyWill Jagy

            104k5102201




            104k5102201












            • $begingroup$
              so many master here, i am lucky!
              $endgroup$
              – tangyao
              Dec 24 '18 at 15:26


















            • $begingroup$
              so many master here, i am lucky!
              $endgroup$
              – tangyao
              Dec 24 '18 at 15:26
















            $begingroup$
            so many master here, i am lucky!
            $endgroup$
            – tangyao
            Dec 24 '18 at 15:26




            $begingroup$
            so many master here, i am lucky!
            $endgroup$
            – tangyao
            Dec 24 '18 at 15:26











            1












            $begingroup$

            This is a particular case of the so-called Mathews diophantine equation, 1889 (or sometimes "cubic Pell") $(M_{m,n})∶ x^3+my^3+m^2 z^3-3mxyz=n$, where $m,n in mathbf Z$. It can be put under a more tractable form by factorizing a certain « normic » expression in the ring $mathbf Z[mu,omega]$, where $omega$ is a primitive cubic root of $1$ and $mu$ is the real cubic root of $m$. Precisely, the splitting field of $X^3-m$ is $mathbf Q (mu,omega)$ whose Galois group is $S_3$, generated by $tau: mu to mu,omega to omega^2$, and $sigma: omega to omega, mu to omegamu$. If 𝒩 is the norm (= product of conjugates) in $mathbf Q(μ,ω)/mathbf Q(mu)$, then 𝒩$(x+yμ+zμ^2)=(x+yμ+zμ^2)(x+yωμ+zω^2 μ^2)(x+yω^2 μ+zωμ^2)=$ $x^3+my^3+m^2 z^3-3mxyz$ (using $1+ω+ω^2=0$), hence eq. $(M_{m,n})$ is simply equivalent to 𝒩$(x+yμ+zμ^2)=n$. The function 𝒩 being multiplicative, we are reduced to solving the case $(M_{m,p})$, where $p$ is a prime number, and the case $(M_{m,pm 1})$.



            Your problem belongs to the case to $(M_{m,pm 1})$, with $m=3$. The equation 𝒩$(x+yμ+zμ^2)=pm 1$ means simply that $x+yμ+zμ^2 in mathbf Z[mu]$ is actually a unit. The ring of integers of $mathbf Q (mu)$ is classically known, see e.g. D. Marcus, "Number Fields", end of chap.2. In particular, for $m=3$, the ring of integers of $mathbf Q (mu)$ is $mathbf Z[mu]$ (which is a PID). We can apply Dirichlet's unit theorem, which states that the group of units has $mathbf Z$-rank $1$. The fundamental unit is determined in exercise 37 (c) of Marcus, chap.5. Note that the resolution of the eq. $(M_{m,pm 1})$ here is quite analogous to that of the quadratic Pell equation (cp. @Tianlalu).



            NB. To my knowledge, a complete solution of $(M_{m,p})$ is not available. A natural approach would be parallel to that exposed in the book of D. Cox, "Primes of the form $x^2+ny^2$", but this requires deep results from CFT.






            share|cite|improve this answer











            $endgroup$













            • $begingroup$
              so many master here, i am lucky!
              $endgroup$
              – tangyao
              Dec 24 '18 at 15:26
















            1












            $begingroup$

            This is a particular case of the so-called Mathews diophantine equation, 1889 (or sometimes "cubic Pell") $(M_{m,n})∶ x^3+my^3+m^2 z^3-3mxyz=n$, where $m,n in mathbf Z$. It can be put under a more tractable form by factorizing a certain « normic » expression in the ring $mathbf Z[mu,omega]$, where $omega$ is a primitive cubic root of $1$ and $mu$ is the real cubic root of $m$. Precisely, the splitting field of $X^3-m$ is $mathbf Q (mu,omega)$ whose Galois group is $S_3$, generated by $tau: mu to mu,omega to omega^2$, and $sigma: omega to omega, mu to omegamu$. If 𝒩 is the norm (= product of conjugates) in $mathbf Q(μ,ω)/mathbf Q(mu)$, then 𝒩$(x+yμ+zμ^2)=(x+yμ+zμ^2)(x+yωμ+zω^2 μ^2)(x+yω^2 μ+zωμ^2)=$ $x^3+my^3+m^2 z^3-3mxyz$ (using $1+ω+ω^2=0$), hence eq. $(M_{m,n})$ is simply equivalent to 𝒩$(x+yμ+zμ^2)=n$. The function 𝒩 being multiplicative, we are reduced to solving the case $(M_{m,p})$, where $p$ is a prime number, and the case $(M_{m,pm 1})$.



            Your problem belongs to the case to $(M_{m,pm 1})$, with $m=3$. The equation 𝒩$(x+yμ+zμ^2)=pm 1$ means simply that $x+yμ+zμ^2 in mathbf Z[mu]$ is actually a unit. The ring of integers of $mathbf Q (mu)$ is classically known, see e.g. D. Marcus, "Number Fields", end of chap.2. In particular, for $m=3$, the ring of integers of $mathbf Q (mu)$ is $mathbf Z[mu]$ (which is a PID). We can apply Dirichlet's unit theorem, which states that the group of units has $mathbf Z$-rank $1$. The fundamental unit is determined in exercise 37 (c) of Marcus, chap.5. Note that the resolution of the eq. $(M_{m,pm 1})$ here is quite analogous to that of the quadratic Pell equation (cp. @Tianlalu).



            NB. To my knowledge, a complete solution of $(M_{m,p})$ is not available. A natural approach would be parallel to that exposed in the book of D. Cox, "Primes of the form $x^2+ny^2$", but this requires deep results from CFT.






            share|cite|improve this answer











            $endgroup$













            • $begingroup$
              so many master here, i am lucky!
              $endgroup$
              – tangyao
              Dec 24 '18 at 15:26














            1












            1








            1





            $begingroup$

            This is a particular case of the so-called Mathews diophantine equation, 1889 (or sometimes "cubic Pell") $(M_{m,n})∶ x^3+my^3+m^2 z^3-3mxyz=n$, where $m,n in mathbf Z$. It can be put under a more tractable form by factorizing a certain « normic » expression in the ring $mathbf Z[mu,omega]$, where $omega$ is a primitive cubic root of $1$ and $mu$ is the real cubic root of $m$. Precisely, the splitting field of $X^3-m$ is $mathbf Q (mu,omega)$ whose Galois group is $S_3$, generated by $tau: mu to mu,omega to omega^2$, and $sigma: omega to omega, mu to omegamu$. If 𝒩 is the norm (= product of conjugates) in $mathbf Q(μ,ω)/mathbf Q(mu)$, then 𝒩$(x+yμ+zμ^2)=(x+yμ+zμ^2)(x+yωμ+zω^2 μ^2)(x+yω^2 μ+zωμ^2)=$ $x^3+my^3+m^2 z^3-3mxyz$ (using $1+ω+ω^2=0$), hence eq. $(M_{m,n})$ is simply equivalent to 𝒩$(x+yμ+zμ^2)=n$. The function 𝒩 being multiplicative, we are reduced to solving the case $(M_{m,p})$, where $p$ is a prime number, and the case $(M_{m,pm 1})$.



            Your problem belongs to the case to $(M_{m,pm 1})$, with $m=3$. The equation 𝒩$(x+yμ+zμ^2)=pm 1$ means simply that $x+yμ+zμ^2 in mathbf Z[mu]$ is actually a unit. The ring of integers of $mathbf Q (mu)$ is classically known, see e.g. D. Marcus, "Number Fields", end of chap.2. In particular, for $m=3$, the ring of integers of $mathbf Q (mu)$ is $mathbf Z[mu]$ (which is a PID). We can apply Dirichlet's unit theorem, which states that the group of units has $mathbf Z$-rank $1$. The fundamental unit is determined in exercise 37 (c) of Marcus, chap.5. Note that the resolution of the eq. $(M_{m,pm 1})$ here is quite analogous to that of the quadratic Pell equation (cp. @Tianlalu).



            NB. To my knowledge, a complete solution of $(M_{m,p})$ is not available. A natural approach would be parallel to that exposed in the book of D. Cox, "Primes of the form $x^2+ny^2$", but this requires deep results from CFT.






            share|cite|improve this answer











            $endgroup$



            This is a particular case of the so-called Mathews diophantine equation, 1889 (or sometimes "cubic Pell") $(M_{m,n})∶ x^3+my^3+m^2 z^3-3mxyz=n$, where $m,n in mathbf Z$. It can be put under a more tractable form by factorizing a certain « normic » expression in the ring $mathbf Z[mu,omega]$, where $omega$ is a primitive cubic root of $1$ and $mu$ is the real cubic root of $m$. Precisely, the splitting field of $X^3-m$ is $mathbf Q (mu,omega)$ whose Galois group is $S_3$, generated by $tau: mu to mu,omega to omega^2$, and $sigma: omega to omega, mu to omegamu$. If 𝒩 is the norm (= product of conjugates) in $mathbf Q(μ,ω)/mathbf Q(mu)$, then 𝒩$(x+yμ+zμ^2)=(x+yμ+zμ^2)(x+yωμ+zω^2 μ^2)(x+yω^2 μ+zωμ^2)=$ $x^3+my^3+m^2 z^3-3mxyz$ (using $1+ω+ω^2=0$), hence eq. $(M_{m,n})$ is simply equivalent to 𝒩$(x+yμ+zμ^2)=n$. The function 𝒩 being multiplicative, we are reduced to solving the case $(M_{m,p})$, where $p$ is a prime number, and the case $(M_{m,pm 1})$.



            Your problem belongs to the case to $(M_{m,pm 1})$, with $m=3$. The equation 𝒩$(x+yμ+zμ^2)=pm 1$ means simply that $x+yμ+zμ^2 in mathbf Z[mu]$ is actually a unit. The ring of integers of $mathbf Q (mu)$ is classically known, see e.g. D. Marcus, "Number Fields", end of chap.2. In particular, for $m=3$, the ring of integers of $mathbf Q (mu)$ is $mathbf Z[mu]$ (which is a PID). We can apply Dirichlet's unit theorem, which states that the group of units has $mathbf Z$-rank $1$. The fundamental unit is determined in exercise 37 (c) of Marcus, chap.5. Note that the resolution of the eq. $(M_{m,pm 1})$ here is quite analogous to that of the quadratic Pell equation (cp. @Tianlalu).



            NB. To my knowledge, a complete solution of $(M_{m,p})$ is not available. A natural approach would be parallel to that exposed in the book of D. Cox, "Primes of the form $x^2+ny^2$", but this requires deep results from CFT.







            share|cite|improve this answer














            share|cite|improve this answer



            share|cite|improve this answer








            edited Dec 24 '18 at 9:34

























            answered Dec 24 '18 at 9:29









            nguyen quang donguyen quang do

            8,9891724




            8,9891724












            • $begingroup$
              so many master here, i am lucky!
              $endgroup$
              – tangyao
              Dec 24 '18 at 15:26


















            • $begingroup$
              so many master here, i am lucky!
              $endgroup$
              – tangyao
              Dec 24 '18 at 15:26
















            $begingroup$
            so many master here, i am lucky!
            $endgroup$
            – tangyao
            Dec 24 '18 at 15:26




            $begingroup$
            so many master here, i am lucky!
            $endgroup$
            – tangyao
            Dec 24 '18 at 15:26


















            draft saved

            draft discarded




















































            Thanks for contributing an answer to Mathematics Stack Exchange!


            • Please be sure to answer the question. Provide details and share your research!

            But avoid



            • Asking for help, clarification, or responding to other answers.

            • Making statements based on opinion; back them up with references or personal experience.


            Use MathJax to format equations. MathJax reference.


            To learn more, see our tips on writing great answers.




            draft saved


            draft discarded














            StackExchange.ready(
            function () {
            StackExchange.openid.initPostLogin('.new-post-login', 'https%3a%2f%2fmath.stackexchange.com%2fquestions%2f3050178%2fprove-x33y39z3-9xyz-1-has-infinity-integers-solutions%23new-answer', 'question_page');
            }
            );

            Post as a guest















            Required, but never shown





















































            Required, but never shown














            Required, but never shown












            Required, but never shown







            Required, but never shown

































            Required, but never shown














            Required, but never shown












            Required, but never shown







            Required, but never shown







            Popular posts from this blog

            To store a contact into the json file from server.js file using a class in NodeJS

            Redirect URL with Chrome Remote Debugging Android Devices

            Dieringhausen